Difference between revisions of "2022 USAMO Problems/Problem 2"
(Created page with "==Problem== Let <math>b\geq2</math> and <math>w\geq2</math> be fixed integers, and <math>n=b+w</math>. Given are <math>2b</math> identical black rods and <math>2w</math> ident...") |
(→Problem) |
||
Line 1: | Line 1: | ||
==Problem== | ==Problem== | ||
− | Let <math>b\geq2</math> and <math>w\geq2</math> be fixed integers, and <math>n=b+w</math>. Given are <math>2b</math> identical black rods and <math>2w</math> identical white rods, each of side length 1. | + | Let <math>b\geq2</math> and <math>w\geq2</math> be fixed integers, and <math>n=b+w</math>. Given are <math>2b</math> identical black rods and <math>2w</math> identical white rods, each of side length <math>1</math>. |
− | We assemble a regular <math>2n | + | We assemble a regular <math>2n</math>-gon using these rods so that parallel sides are the same color. Then, a convex <math>2b</math>-gon <math>B</math> is formed by translating the black rods, and a convex <math>2w</math>-gon <math>W</math> is formed by translating the white rods. An example of one way of doing the assembly when <math>b=3</math> and <math>w=2</math> is shown below, as well as the resulting polygons <math>B</math> and <math>W</math>. |
+ | |||
+ | <asy> | ||
+ | size(10cm); | ||
+ | real w = 2*Sin(18); | ||
+ | real h = 0.10 * w; | ||
+ | real d = 0.33 * h; | ||
+ | picture wht; | ||
+ | picture blk; | ||
+ | |||
+ | draw(wht, (0,0)--(w,0)--(w+d,h)--(-d,h)--cycle); | ||
+ | fill(blk, (0,0)--(w,0)--(w+d,h)--(-d,h)--cycle, black); | ||
+ | |||
+ | // draw(unitcircle, blue+dotted); | ||
+ | |||
+ | // Original polygon | ||
+ | add(shift(dir(108))*blk); | ||
+ | add(shift(dir(72))*rotate(324)*blk); | ||
+ | add(shift(dir(36))*rotate(288)*wht); | ||
+ | add(shift(dir(0))*rotate(252)*blk); | ||
+ | add(shift(dir(324))*rotate(216)*wht); | ||
+ | |||
+ | add(shift(dir(288))*rotate(180)*blk); | ||
+ | add(shift(dir(252))*rotate(144)*blk); | ||
+ | add(shift(dir(216))*rotate(108)*wht); | ||
+ | add(shift(dir(180))*rotate(72)*blk); | ||
+ | add(shift(dir(144))*rotate(36)*wht); | ||
+ | |||
+ | // White shifted | ||
+ | real Wk = 1.2; | ||
+ | pair W1 = (1.8,0.1); | ||
+ | pair W2 = W1 + w*dir(36); | ||
+ | pair W3 = W2 + w*dir(108); | ||
+ | pair W4 = W3 + w*dir(216); | ||
+ | path Wgon = W1--W2--W3--W4--cycle; | ||
+ | draw(Wgon); | ||
+ | pair WO = (W1+W3)/2; | ||
+ | transform Wt = shift(WO)*scale(Wk)*shift(-WO); | ||
+ | draw(Wt * Wgon); | ||
+ | label("$W$", WO); | ||
+ | /* | ||
+ | draw(W1--Wt*W1); | ||
+ | draw(W2--Wt*W2); | ||
+ | draw(W3--Wt*W3); | ||
+ | draw(W4--Wt*W4); | ||
+ | */ | ||
+ | |||
+ | // Black shifted | ||
+ | real Bk = 1.10; | ||
+ | pair B1 = (1.5,-0.1); | ||
+ | pair B2 = B1 + w*dir(0); | ||
+ | pair B3 = B2 + w*dir(324); | ||
+ | pair B4 = B3 + w*dir(252); | ||
+ | pair B5 = B4 + w*dir(180); | ||
+ | pair B6 = B5 + w*dir(144); | ||
+ | path Bgon = B1--B2--B3--B4--B5--B6--cycle; | ||
+ | pair BO = (B1+B4)/2; | ||
+ | transform Bt = shift(BO)*scale(Bk)*shift(-BO); | ||
+ | fill(Bt * Bgon, black); | ||
+ | fill(Bgon, white); | ||
+ | label("$B$", BO); | ||
+ | </asy> | ||
Prove that the difference of the areas of <math>B</math> and <math>W</math> depends only on the numbers <math>b</math> and <math>w</math>, and not on how the <math>2n</math>-gon was assembled. | Prove that the difference of the areas of <math>B</math> and <math>W</math> depends only on the numbers <math>b</math> and <math>w</math>, and not on how the <math>2n</math>-gon was assembled. | ||
+ | |||
==Solution== | ==Solution== | ||
[WIP] | [WIP] |
Revision as of 18:40, 26 May 2022
Problem
Let and be fixed integers, and . Given are identical black rods and identical white rods, each of side length .
We assemble a regular -gon using these rods so that parallel sides are the same color. Then, a convex -gon is formed by translating the black rods, and a convex -gon is formed by translating the white rods. An example of one way of doing the assembly when and is shown below, as well as the resulting polygons and .
Prove that the difference of the areas of and depends only on the numbers and , and not on how the -gon was assembled.
Solution
[WIP]
See also
2022 USAMO (Problems • Resources) | ||
Preceded by Problem 1 |
Followed by Problem 3 | |
1 • 2 • 3 • 4 • 5 • 6 | ||
All USAMO Problems and Solutions |
The problems on this page are copyrighted by the Mathematical Association of America's American Mathematics Competitions.